Difference between revisions of "2000 AMC 12 Problems/Problem 13"

m (ambiguity fix)
(duplicate)
Line 1: Line 1:
 +
{{duplicate|[[2000 AMC 12 Problems|2000 AMC 12 #13]] and [[2000 AMC 10 Problems|2000 AMC 10 #22]]}}
 +
 
== Problem ==
 
== Problem ==
 
One morning each member of Angela’s family drank an 8-ounce mixture of coffee with milk. The amounts of coffee and milk varied from cup to cup, but were never zero. Angela drank a quarter of the total amount of milk and a sixth of the total amount of coffee. How many people are in the family?
 
One morning each member of Angela’s family drank an 8-ounce mixture of coffee with milk. The amounts of coffee and milk varied from cup to cup, but were never zero. Angela drank a quarter of the total amount of milk and a sixth of the total amount of coffee. How many people are in the family?
Line 11: Line 13:
 
== See also ==
 
== See also ==
 
{{AMC12 box|year=2000|num-b=12|num-a=14}}
 
{{AMC12 box|year=2000|num-b=12|num-a=14}}
 +
{{AMC10 box|year=2000|num-b=21|num-a=23}}
  
 
[[Category:Introductory Number Theory Problems]]
 
[[Category:Introductory Number Theory Problems]]

Revision as of 00:05, 27 November 2011

The following problem is from both the 2000 AMC 12 #13 and 2000 AMC 10 #22, so both problems redirect to this page.

Problem

One morning each member of Angela’s family drank an 8-ounce mixture of coffee with milk. The amounts of coffee and milk varied from cup to cup, but were never zero. Angela drank a quarter of the total amount of milk and a sixth of the total amount of coffee. How many people are in the family?

$\text {(A)}\ 3 \qquad \text {(B)}\ 4 \qquad \text {(C)}\ 5 \qquad \text {(D)}\ 6 \qquad \text {(E)}\ 7$

Solution

Let $a$ be the total amount of coffee, $b$ of milk, and $n$ the number of people in the family. Then each person drinks the same total amount of coffee and milk (8 ounces), so \[\left(\frac{a}{6} + \frac{b}{4}\right)n = a + b\] Regrouping, we get $2a(6-n)=3b(n-4)$. Since both $a,b$ are positive, it follows that $6-n,n-4$ are also positive, which is only possible when $n = 5\ \mathrm{(C)}$.

See also

2000 AMC 12 (ProblemsAnswer KeyResources)
Preceded by
Problem 12
Followed by
Problem 14
1 2 3 4 5 6 7 8 9 10 11 12 13 14 15 16 17 18 19 20 21 22 23 24 25
All AMC 12 Problems and Solutions
2000 AMC 10 (ProblemsAnswer KeyResources)
Preceded by
Problem 21
Followed by
Problem 23
1 2 3 4 5 6 7 8 9 10 11 12 13 14 15 16 17 18 19 20 21 22 23 24 25
All AMC 10 Problems and Solutions